Difference between revisions of "2000 AIME I Problems/Problem 7"
Hashtagmath (talk | contribs) |
m |
||
(3 intermediate revisions by 2 users not shown) | |||
Line 1: | Line 1: | ||
== Problem == | == Problem == | ||
Suppose that <math>x,</math> <math>y,</math> and <math>z</math> are three positive numbers that satisfy the equations <math>xyz = 1,</math> <math>x + \frac {1}{z} = 5,</math> and <math>y + \frac {1}{x} = 29.</math> Then <math>z + \frac {1}{y} = \frac {m}{n},</math> where <math>m</math> and <math>n</math> are [[relatively prime]] positive integers. Find <math>m + n</math>. | Suppose that <math>x,</math> <math>y,</math> and <math>z</math> are three positive numbers that satisfy the equations <math>xyz = 1,</math> <math>x + \frac {1}{z} = 5,</math> and <math>y + \frac {1}{x} = 29.</math> Then <math>z + \frac {1}{y} = \frac {m}{n},</math> where <math>m</math> and <math>n</math> are [[relatively prime]] positive integers. Find <math>m + n</math>. | ||
− | |||
− | |||
− | |||
==Solution 1== | ==Solution 1== | ||
Line 55: | Line 52: | ||
==Solution 5== | ==Solution 5== | ||
Get rid of the denominators in the second and third equations to get <math>xz-5z=-1</math> and <math>xy-29x=-1</math>. Then, since <math>xyz=1</math>, we have <math>\tfrac 1y-5z=-1</math> and <math>\tfrac 1z-29x=-1</math>. Then, since we know that <math>\tfrac 1z+x=5</math>, we can subtract these two equations to get that <math>30x=6\implies x=5</math>. The result follows that <math>z=\tfrac 5{24}</math> and <math>y=24</math>, so <math>z+\tfrac 1y=\tfrac 1{24}+\tfrac 5{24}=\tfrac 14</math>, and the requested answer is <math>1+4=\boxed{005}.</math> | Get rid of the denominators in the second and third equations to get <math>xz-5z=-1</math> and <math>xy-29x=-1</math>. Then, since <math>xyz=1</math>, we have <math>\tfrac 1y-5z=-1</math> and <math>\tfrac 1z-29x=-1</math>. Then, since we know that <math>\tfrac 1z+x=5</math>, we can subtract these two equations to get that <math>30x=6\implies x=5</math>. The result follows that <math>z=\tfrac 5{24}</math> and <math>y=24</math>, so <math>z+\tfrac 1y=\tfrac 1{24}+\tfrac 5{24}=\tfrac 14</math>, and the requested answer is <math>1+4=\boxed{005}.</math> | ||
+ | |||
+ | ==Solution 6== | ||
+ | Rewrite the equations in terms of x. | ||
+ | |||
+ | <math>x+\frac{1}{z}=5</math> becomes <math>z=\frac{1}{x+5}</math>. | ||
+ | |||
+ | <math>y+\frac{1}{x}=29</math> becomes <math>y=29-\frac{1}{x}</math> | ||
+ | |||
+ | Now express <math>xyz=1</math> in terms of x. | ||
+ | |||
+ | <math>\frac{1}{5-x}\cdot(29-\frac{1}{x})\cdot x=1</math>. | ||
+ | |||
+ | This evaluates to <math>29x-1=5-x</math>, giving us <math>x=\frac{1}{5}</math>. We can now plug x into the other equations to get <math>y=24</math> and <math>z=\frac{5}{24}</math>. | ||
+ | |||
+ | Therefore, <math>z+\frac{1}{y}=\frac{5}{24}+\frac{1}{24}=\frac{6}{24}=\frac{1}{4}</math>. | ||
+ | |||
+ | <math>1+4=\boxed{5}</math>, and we are done. | ||
+ | ~MC413551 | ||
== See also == | == See also == |
Latest revision as of 17:16, 27 September 2024
Contents
Problem
Suppose that and are three positive numbers that satisfy the equations and Then where and are relatively prime positive integers. Find .
Solution 1
We can rewrite as .
Substituting into one of the given equations, we have
We can substitute back into to obtain
We can then substitute once again to get Thus, , so .
Solution 2
Let .
Thus . So .
Solution 3
Since , so . Also, by the second equation. Substitution gives , , and , so the answer is 4+1 which is equal to .
Solution 4
(Hybrid between 1/2)
Because and . Substituting and factoring, we get , , and . Multiplying them all together, we get, , but is , and by the Identity property of multiplication, we can take it out. So, in the end, we get . And, we can expand this to get , and if we make a substitution for , and rearrange the terms, we get This will be important.
Now, lets add the 3 equations , and . We use the expand the Left hand sides, then, we add the equations to get Notice that the LHS of this equation matches the LHS equation that I said was important. So, the RHS of both equations are equal, and thus We move all constant terms to the right, and all linear terms to the left, to get , so which gives an answer of
-AlexLikeMath
Solution 5
Get rid of the denominators in the second and third equations to get and . Then, since , we have and . Then, since we know that , we can subtract these two equations to get that . The result follows that and , so , and the requested answer is
Solution 6
Rewrite the equations in terms of x.
becomes .
becomes
Now express in terms of x.
.
This evaluates to , giving us . We can now plug x into the other equations to get and .
Therefore, .
, and we are done. ~MC413551
See also
2000 AIME I (Problems • Answer Key • Resources) | ||
Preceded by Problem 6 |
Followed by Problem 8 | |
1 • 2 • 3 • 4 • 5 • 6 • 7 • 8 • 9 • 10 • 11 • 12 • 13 • 14 • 15 | ||
All AIME Problems and Solutions |
The problems on this page are copyrighted by the Mathematical Association of America's American Mathematics Competitions.